Download as docx, pdf, or txt
Download as docx, pdf, or txt
You are on page 1of 18

“PENGANTAR FISIKA KUANTUM”

CHAPTER 2
THE SCHRODINGER EQUATION

Dosen Pengampu:
Prof. Dr. I Wayan Santyasa, M.Si.,

Oleh :
Ni Putu Indah Pratiwi 1713021029/ VI B

PROGRAM STUDI PENDIDIKAN FISIKA


JURUSAN FISIKA DAN PENGAJARAN IPA
FAKULTAS MATEMATIKA DAN ILMU PENGETAHUAN ALAM
UNIVERSITAS PENDIDIKAN GANESHA
SINGARAJA
2020
1. Problem 2.6 Consider the Gaussian distribution
2

ρ ( x )= Ae− λ(x−a)

where A , a , dan λ are constant (look up any integrals you need).

a) Use Equation 2.16 to determine A.


b) Find (x) (x2) and σ
c) Sketch the graph of p(x )

Solution:

a) Use Equation 2.16 to determine A.


Answer:
Gunakan persamaan 2.16,
+∞

∫ ρ ( x ) dx=1
−∞
2

ρ ( x )= Ae− λ(x−a) sehingga


+∞
2

∫ Ae−λ(x−a) dx= 1
−∞

+∞
2

A ∫ e−λ( x−a ) dx= 1


−∞

u = √ λ(¿ x−a)¿ dan du = √ λdx didapat :

+∞
2
du
A ∫ e−u =1
−∞ √λ
+∞
A 2

∫ e−u du= 1
λ
√ −∞
+∞
2

Standar Gaussian integral : ∫ e−x dx = √ π


−∞

pengintegralan di atas menjadi,


A π
√ π =1 ⥤ A= √
√λ √λ
π λ
1= A
√ λ
⥤ A=
π √
π (− λ ( x−a) )

2

ρ ( x )= e
λ

b) Find (x) (x2) and σ


Answer:
Dari persamaan 2.17 atau 2.18..
+∞
⟨ x ⟩= ∫ xρ ( x ) dx dan
−∞

+∞
⟨ x 2 ⟩= ∫ x 2 ρ ( x ) dx
−∞

nilai ⟨ x ⟩ :
+∞
⟨ x ⟩= √ λ ∫ xe− λ(x−a) dx
2

√ π −∞
Misal u = √ λ(¿ x−a)¿, du = √ λdx

+∞
⟨ x ⟩= √ λ ∫ xe−u du
2

√ π −∞ √λ
Misal x = u + a, maka
+∞
⟨ x ⟩= 1
2

∫ (u+a) e−u du
√π −∞

+∞ +∞
⟨ x ⟩= 1 ∫ ue−u du+ ∫ ae−u du
2 2

√ π −∞ −∞

a adalah konstanta, maka integral ini diselesaikan dengan fungsi ganjil genap.
2
u adalah fungsi genap dan e−u fungsi ganjil.

misal w = u2maka dw = 2udu, maka u tidak di kalikan dengan fungsi eksponensialnya.


batas integralnya berubah menjadi w ( ∞ )=∞ 2=∞ dan w(-∞ ¿= (∞ ¿ ¿2 = ∞.

+∞
⟨ x ⟩=0+ a ∫ ae−u du
2

√ π −∞
a
⟨ x ⟩=0+ √ π =a
√π
nilai ⟨ x 2 ⟩
dari persamaan 2.18 yaitu f (x) = x2 , maka
+∞

⟨ x ⟩ = ¿ √ λ ∫ x 2 e− λ(x−a) dx
2
2

√ π −∞
+∞
⟨ x 2 ⟩= √ λ ∫ (u+ a)2 e− λu du
2

√ π −∞
+∞
⟨ x 2 ⟩= √ λ ∫ (u2 +2 ua+a ¿ ¿ 2)e−λu du ¿
2

√ π −∞
+∞ +∞ +∞
2

√ π −∞ [
⟨ x ⟩= √ λ ∫ u2 e−λu du+2 a ∫ ue−λu du+a 2 ∫ e− λu du
−∞ −∞
2 2 2

]
Dua integral terakhir sama ketika kita mengkalkulasikan dengan nilai x rata-rata, kita
dapatkan
+∞

√ π −∞ [
⟨ x 2 ⟩= √ λ ∫ u2 e−λu du+2 a .0+a 2 √ π
√λ
2

]
+∞

∫ e−λx dx = √√πλ
2

Untuk integral pertama , dimana


−∞

2
Dengan mengkalkulasikan integral x 2 e− λx yang menjadi turunan negative dari
Gaussian

d − λx 2 2

e =−x 2 e− λx

+∞
d
∫ e−λu du= ddλ √π
2

d λ −∞ √λ
+∞
d −λu −1 √ π 2

∫ e du = 2 3
−∞ dλ λ2

+∞
2 1 √π
∫ x e−λu du = 2 3
2
−∞ λ

⟨ x 2 ⟩= √ λ √ π3 +0+ a2 √ π
√π 2 [ √λ
2λ ]
2
1
⟨ x 2 ⟩= +a

Nilai standar deviasi,
2
1 1
σ = ⟨ x2 ⟩− ⟨ x ⟩ =
√ 2

√ 2λ
+a −a2 =
2λ √
c) Membuat grafik p(x )
x = 0, ± a , 2 a
x  ±∞

2. Problem 2.7 At time t = 0 a particle is represented by the wave function


Ax

{
, if 0 ≤ x ≤ a ,
a
Ψ ( x , 0 )= A ( b−x )
,if a ≤ x ≥b ,
( b−a )
0 , o therwise ,

Where A, a and B are constants.


a) Normalize Ψ (what is find A in terms of a and b)
b) Sketch Ψ ( x , 0 ) as a function of x
c) Where is the particle most likely to be found, at t=0 ?
d) What is the probability of finding the particle to the left of a ? Chek your result in
the limiting cases b=a∧b=2 a.
e) What is the expectation value of x?

Solution:

a) Normalizing means integrating |Ψ |2over all space, but since ψ is zero except over
[0,b], we need only perform the integral over that interval. We will assume A may be
complex, but will presume a and b to be real.
| A|2
a
| A|2
b
2
1=
a 2 ∫ x dx + 2 ∫ ( b−x )2 dx
0 ( b−a ) a
a b
1 x3
¿| A| 2
a 3
2
[ ( )| +
1
0 ( b−a )
2
3
3
−( b−x )
( )| ]
a

a b−a 2b
¿| A|
2
[ 3
+
3 ]
=| A|
3

3
A=
√ b

b) In the end, A is purely real, but best to be on the safe side. The wave function is
linearly increasing from 0 to a, going from Ψ ( 0 ) =0 to Ψ ( a ) =A, and linearly
decreasing from a to b, going from Ψ ( a ) =A to . Ψ ( b ) =A .A sketch is left as an
exercise to the reader.

c) The particle is most likely to be found where |ψ| 2 has a maximum, which in this case
means the place where ψ itself is maximum. From the given form (or your sketch), the
maximum is clearly at x=a.
d) The probability of finding the particle to the left of a is given by integrating the
probability density from −∞ to a:

a
|A|2
a
3 x3 a
P ( ¿ [ −∞ ,a ] ) = 2 ∫ x dx= 2
a 0
2
=
ba 3 0 b |
In the limiting case thatb=a,P=1 In this case the wavefunction is zero for x >a, so
the particle has no where else to be. In the limiting caseb=2 a, the wavefunction is
defined over two symmetric intervals, with x=a right in the middle. By symmetry, the

1
particle must be to the left of a half of the time, and we indeed find P=
2
e) The expectation value is found by integrating x|ψ| 2 over all space, or over [0,b] since
ψ is zero elsewhere.
2 a 2 b
2 |A| 3 | A|
⟨ x ⟩=∫ x|Ψ | dx=
a 2 ∫ x dx+ 2 ∫ ( b−x )2 dx
0 ( b−a ) a

a b

¿
[ ( )|
3 1 x4
b a2 4
+
1
0 (b−a)
2
b 2 x
2
2
x3 x4
−2 b +(
3 4 )| ]
a

3 8 b4 2
2 2 8a b
¿
4 b ( b−a ) 2 [
2 2 4
a (b−a) +2 b −
3
−2 a b +
3
−a4 ]
3 b3 2 2 2 3 1
¿ 2( −a b + a b = 2)( b3 −3 a2 b +2 a3 )= 2 a+b
4 b ( b−a ) 3 3 4 b ( b−a ) 4

Wolfram Alpha will tell you that b 3−3 a2 b+2 a3 factors to( a−b )2 (2 a+ b) if you didn’t
notice. The last step is only valid if a ≠ bwhich seems sensible based on the definition
of the wavefunction given.

3. Problem 2.8 Consider the wave function

Ψ ( x , t )= Ae−⋋|x|e−iωt

where A,⋋ and ω are positive real constants

a) Normalize Ψ
b) Determine the expectation values of x and x2
c) Find the standard deviation of x. Sketch the graph of |Ψ |2 as a function of x, and
mark the points (⟨ x ⟩+σ ) and (⟨ x ⟩− σ ), to illustrate the sense in which σ represents the
“spread” in x. What is the probability that the particle would be found outside this
range?
Solution:
a) Normalize Ψ
2
ρ ( x )=|Ψ ( x ,t )| =Ψ ¿ Ψ
Ψ =Ae−⋋|x| e−iωt
Ψ ¿ = Ae−⋋|x|+iωt
ρ ( x )= A2 e(−2 ⋋|x|)
+∞
1= ∫ ρ ( x ) dx
−∞

+∞
1
A =∫ e(−2 ⋋|x|) dx=
−2

−∞ ⋋
A=√ ⋋
ρ ( x )=⋋ e(−2 ⋋|x|)
Ψ ( x , t )=√ ⋋ e (−⋋|x|−iωt )
b) Determine the expectation values of x and x2
+∞ +∞
( x )= ∫ xρ ( x ) dx= ∫ x ⋋ e(−2 ⋋|x|) dx=0
−∞ −∞

+∞ +∞
( x 2 )= ∫ x 2 ρ ( x ) dx=∫ x 2 ⋋ e (−2 ⋋|x|) dx= 1 2
−∞ −∞ 2⋋
c) Find the standard deviation of x. Sketch the graph of |Ψ |2 as a function of x, and mark
the points ((x) +σ ) and ((x) − σ ), to illustrate the sense in which σ represents the
“spread” in x. What is the probability that the particle would be found outside this
range?
1
σ =√ ( x 2) −(x )2=√ ( x 2 )−0=( x 2 )=
2 ⋋2

The probability of finding the particle outside this range is the complement of the
probability of finding the particle inside this range, which is given by
'
ρ [ ( x )−σ < x < ( x ) +σ ] =1−ρ [ ( x )−σ < x< ( x ) + σ ]
( x )+ σ

¿ 1− ∫ ρ ( x )d x
( x ) −σ
−1 (−2 ⋋| x|)
(2 ⋋¿¿2) ⋋ e dx¿

¿ 1− ∫ ¿
2 −1
−(2 ⋋ )
−1

¿ 1−( 1e (−⋋ )
)
−1

¿ e (−⋋ )

4. Problem 2.9 Let Pab (t ) be the probability of finding the particle in the range

( a<x <b ) , at time t


a) Show that
dPab
= J ( a , t )− J ( b , t )
dt
Where
¿
iℏ ∂ψ ¿ ∂ψ
J ( x ,t ) = ψ
2m ∂ x
−ψ
∂x ( )
What are the units of J (x,t)? [J is called the probability current, because it tells you
the rate at which probability is “flowimg” past the point x. If Pab (t) is increasing, then
more probability is flowing into the region at one end than flow out other].
b) Find the probability current for the wave function in the previous problem. (This is
not very pithy example, I’m afraid; we’ll encounter some more substantial ones in due
course).

Solution:

a) Jawaban
dPab b
= ∫|ψ ( x ,t )|2 dx
dt a
dPab b ∂ 2
= ∫ |ψ| dx
dt a
∂t
Dimana
¿
∂ |ψ 2|= ∂ | i ℏ ψ ∂ψ − ψ ¿ ∂ ψ |
∂t ∂ x 2m ∂ x ∂x ( )
Bandingkan dengan
∂|ψ|2
=− ∂ J ( x ,t )
∂t ∂t
b
∂ Pab
=−∫ ∂ J ( x ,t ) dx
∂t a ∂x
=−J ( x,t )|ba
= J ( a,t )− j ( b,t )
b) Jawaban
−a [ ( mx 2 /ℏ + it ) ]
ψ ( x ,t ) = Ae
= f ( x ) eiat
dimana:
f ( x ) = Ae−amx/ℏ
¿
∂ψ df
ψ = f ( x ) e−iat eiat
∂x dx
df
=f
dx
∂ψ df
= f ( x ) e−iat eiat
¿
ψ
∂x dx
df
=f
dx
J ( x ,t ) =0
¿
2
∫ e−( ax +bx ) dx
Perhatikan perhitungan dalam integral −¿ dengan variabel
b
y = √a x +
[ ] 2a
dim ana
y b
[x= −
√ a 2a ]
2
b y
2
ax + bx = a −
√a 2 a ( ) + b ( √ya − b2a )
y 2 2 yb b2 by b2
=a
a( −
2
+ 2 +
2 a √a 4 a

√a 2a )( )
2 b
=y −
4a
Karena itu
2
b
¿ ¿ − y2 − 1
−( ax2 +bx ) 4a
∫ e dx = ∫ e dy
−¿ −¿ √a
b2 ¿
1 2
= e 4a
∫ e− y dy
√a −¿
b2
1 4a
= e
√a
Maka
2
¿ b
−( ax2 +bx ) 1
∫−¿ e dx = e 4 a
√a
5. Problem 2.10 Suppose you wanted to describe an unstable particle that spontaneously
disintegrates with a “lifetime” τ. In that case the total probability of finding the particle
somewhere should not be constant, but should decrease at (say) an exponential rate:

+∞ −t
2 τ
P ( t ) ≡ ∫ |ψ (x ,t )| dx =e
−∞

A crude way of achieving this result is as follows. In equation 2.24 we tacitly assumed
that V (the potential energy) is real. That is certainly reasonable, but it leads to the
conservation of probability enshrined in equation 2.27. What if we assign to V an
imaginary part:
V =V 0−iτ
Where V 0 is the true potential energy and F is a positive real constant?
a) Show that (in place of equation 2.27) we now get
dP −2 τ
= P
dt ℏ
b) Solve of P ( t ) , and find lifetime of the particle in terms τ .
Solution:

dP −2 τ
a) Show that we now get = P
dt ℏ

dPab d b ¿ b

= ∫ ψ ψ dx=∫ (ψ ¿ ¿ ψ)d x ¿
dt dt a a ∂t

b
∂ ψ¿ ∂ψ
¿∫
a
( ∂t
ψ+ψ ¿
∂t
dx=0 )
According to conservation of probability. We can get an expression for the time
derivative of ψ from the Schrodinger equation with our complex potential:
∂ ψ −ℏ2 ∂2 ψ
iℏ = +(V 0−iτ )ψ
∂t 2 m ∂ x 2
∂ψ iℏ ∂ 2 ψ i i
⟺ = − Vψ− ψ
∂ t 2 m ∂ x2 ℏ τ
∂ψ ¿ −iℏ ∂2 ψ ¿ i i ¿

¿
= 2
− Vψ− ψ
∂t 2m ∂ x ℏ τ
So the time derivative of the probability of the can written as:
d P ab b ∂ψ ¿ ∂ψ
b
τ τ
dt
=∫
a ∂t( ψ +ψ ¿
∂t a ℏ
) ℏ (
dx−∫ ψ ¿ ψ +ψ ¿ ψ dx )
2τ ¿
¿ 0− ψ ψd x


¿ P
ℏ ab
b) Solve of P ( t ) , and find lifetime of the particle in terms τ .
d P ab 2 τ
= P
dt ℏ
t
−2 τ

⟺ P ( t )=P (0)e

Which gives the life time τ =

d ⟨ p⟩
6. Problem 2.12 Calculate . Answer.
dt
d ⟨ p ⟩ ∂V
dt
= ( )
∂x
(This is known as Ehrenfest's theorem; it tells us that expectation values obey Newton's
second law.)
Solution:

∞ ∂Ψ
⟨ p ⟩=−iℏ ∫−∞ Ψ ¿ dx
∂x
d ⟨ p⟩ ∞ ∂Ψ
=−iℏ ∫−∞ Ψ ¿ dx
dt ∂x
¿
¿ ∂ ∂Ψ ∂Ψ ∂ Ψ

=−i ℏ ∫−∞ Ψ [ +
∂t ∂ x ∂t ∂ x
dx ]
From Schrödinger equation:
∂Ψ i ℏ ∂ 2 Ψ i ∂ Ψ¿ i ℏ ∂2 Ψ ¿ i ¿
= 2
−ℏ V Ψ ; =− 2
+ℏ V Ψ
∂t 2 m ∂ x ∂t 2m ∂ x
2 ¿
∂⟨ p⟩
dt

=−i ℏ ∫−∞ Ψ ¿ ∂
∂Ψ
∂t ∂ x [
+−
iℏ ∂ Ψ i
2m ∂x 2
+ℏ V Ψ ¿
∂Ψ
(
∂x
dx
) ]
Note that the wavefunction has continuous second partial derivatives, and therefore the
partial derivatives are commutative (by Schwarz’ theorem)

i ℏ ∂2 Ψ i
[ ]
¿
∂⟨ p⟩ ∂Ψ ¿ ∂Ψ
( )

=−i ℏ ∫−∞ Ψ ∂
¿
+− +ℏ V Ψ dx
∂t ∂ x ∂t 2m ∂x 2 ∂x
2 2

[ ( ]
¿
iℏ ∂ Ψ i iℏ ∂ Ψ i

=−i ℏ ∫−∞ Ψ ∂
¿
∂x 2m ∂x 2
−ℏ V Ψ + −
2m ∂x 2
+ℏ V Ψ
¿ ∂Ψ
∂x
dx )( )
i ℏ ∂ 3 Ψ i ∂V i ℏ ∂2 Ψ i
{[
¿
∂Ψ
)] ( ¿ ∂Ψ
) }

=−i ℏ ∫−∞ Ψ
¿
2m ∂x 3
−ℏ
∂x
Ψ +V
∂x ( +−
2m ∂ x 2
+ℏ V Ψ
∂x
dx

i ℏ ∂2 Ψ
]}
3 ¿
=−i ℏ ∫−∞

{ [(
∂Ψ
∂x
¿ i

i
Ψ − V+ V −
ℏ 2m ∂x 2

¿ iℏ ∂ Ψ
2m ∂ x3

i ∂V
)
ℏ ∂x
ψ dx
] [
i ℏ ∂ Ψ ∂2 Ψ ¿ 3
=−i ℏ ∫−∞

{

2m ∂ x ∂x 2

¿ iℏ ∂ Ψ

2m ∂x 3
i ∂V
−ℏ
∂x
ψ dx
[ ]}
i ℏ ∂3 Ψ i ℏ ∂ Ψ ∂2 Ψ
¿
i ∂V
( )
∞ ∞
=−i ℏ ∫−∞ Ψ ¿ − ( ℏ ∂x )
Ψ dx−i ℏ ∫−∞ Ψ ¿ −
2 m ∂ x3 2 m ∂ x ∂ x 2
dx

ℏ2 ∂3 Ψ ∞ ∂Ψ ∂ 2 Ψ ¿

=∫−∞ Ψ −
¿
( ∂V
∂x
Ψ dx+)2m [ ∞
∫−∞ Ψ ¿

∂ x3
dx−∫−∞ ∂ x ∂ x 2 dx ]
∂3 Ψ ∂2 Ψ
¿
∂Ψ
¿ du= dx dυ= 3 dx υ= 2
Let u=Ψ , then ∂x . Let ∂x , then ∂x .

∂2 Ψ ∂2 Ψ
¿
∂Ψ ∂Ψ
μ= dμ= 2 dx dv= 2 v=
Let ∂ x , then ∂x . Let ∂ x , then ∂x


ℏ2
]}
2
d ⟨ p⟩ ∞ ¿ ∂V ∂2 Ψ ∂Ψ ¿ ∂Ψ ¿ ∂2 Ψ
{( ) ∂Ψ ∂Ψ ¿ ∞
¿∂ Ψ
dt
=∫−∞ Ψ −
∂x
Ψ dx+
2m ( )
Ψ 2
∂x −∞
−∫−∞ 2
∂x ∂x

dx−
∂x ∂x [( )−∞
−∫−∞

∂ x ∂ x2
dx


ℏ2 2

{( ) }

∂V ¿∂ Ψ ∂Ψ ∂Ψ ¿

=∫−∞ Ψ −
∂x
¿
( )
Ψ dx +
2m
Ψ
∂ x2 −∞
− ( ∂x ∂x ) −∞
2
Ψ(x,t) must go to zero as x goes to (±) infinity, so the entire ℏ /2 m term is zero.

d ⟨ p⟩ ∞ ¿ ∂V
dt
=∫−∞ Ψ −
∂x ( )
Ψ dx

∂V
=⟨− ⟩
∂x

7. Problem 2.13 Suppose you add a constant V0 to the potential energy (by "constant" mean
independent of x as well as t). In classical mechanics this doesn't change anything, but
what about quantum mechanics? Show that the wave function picks up a time dependent
phase factor: exp(—i V0t/ħ). What effect does this have on the expectation value of a
dynamical variable?

Solution:
Suppose originally solves the Schrödinger equation without V 0
∂ ψ −ℏ2 ∂2 ψ
iℏ = +V ψ ...(1)
∂t 2 m ∂ x 2
We want to find ψ 0 such that
∂ ψ 0 −ℏ2 ∂2 ψ 0
iℏ = +(V +V 0 )ψ
∂t 2 m ∂ x2
Our claim is that
ψ 0=ψ e−i V 0 t/ℏ

Taking the derivative of the above,


∂ ψ0 ∂ψ i V −i V 0
iℏ
∂t
=iℏ
∂t
e 0 t/ℏ
+iℏ ( ℏ )
+ψ e−i V 0 t /ℏ

∂ψ
we know that i ℏ is from equation (1)
∂t
∂ ψ 0 −ℏ2 ∂2 ψ −iV 0
iℏ
∂t
=(2m ∂ x 2
+V ψ e
iV
) 0 t /ℏ
+iℏ ( ℏ
ψe )
−i V 0 t /ℏ

Since ψ 0 and ψ have the same dependence on x, their spatial derivatives only differ by
the same Factor e i V 0 t /ℏ
that Since ψ 0 and ψ do:
∂2 ψ 0 ∂2 ψ −i V t /ℏ
= 2e 0

∂t ∂x
Substituting this,
∂ ψ 0 −ℏ2 ∂2 ψ iV t /ℏ −i V 0t / ℏ
iℏ = +V ψ e 0
+V 0 ψ e
∂t 2 m ∂ x2
But since we claim ψ 0=ψ e−i V 0 t/ℏ

∂ ψ 0 −ℏ2 ∂2 ψ 0
iℏ = +(V +V 0 )ψ 0
∂t 2 m ∂ x2
Thus, shifting the overall potential energy by a constant amount V 0 just adds a phase
factor e−i V 0 t/ℏ
to the wavefunction. This has no effect on the expectation values
dynamical variables like x and p - since neither the x nor p operator involves the time
variable, the phase factor will always cancel out when we take complex conjugates
and calculate the expectation values. The overall phase ofthe wave function will
cancel out whenever you calculate a measurable quantity.
8. Problem 2.14 A particle of mass m is in the state
m x2

ψ ( x , t ) =A e
−a
[( ) ]

+it

Where A and a are positive real constants


a) Find A.
b) For what potential energy function V(x) does Ψ satisfy the schrodringer equation ?
c) Calculate the expectation values of x, x2 , p, p2.
d) Find σ x and σ p. Is their product consitens with the uncertainty principle?.
Solution:

a) Find A
Answer:
In order to find A, we have to normalize. First, since we have a complex
wavefunction, let’s make sure we get the square right. We will assume A is a positive
real constant for convenience
|ψ|2 =ψ ¿ ψ=¿
If A were complex, we could always absorb the imaginary part into the
exponential, which would make it an offset in the t term. That amounts to a
choice of when to zero your clock, which we can always do, so we lose
nothing by making the assumption that A is real.

You didn’t forget to do the complex conjugate, right? Now we can normalize,
remembering that
2
∞ ∞ −2 am x
πℏ
2
1= ∫ |ψ| dx= ∫ A e
−∞ −∞
2 ℏ
dx=¿ A 2
√ 2 am
¿

2 am
A=

4

πℏ

b) For what potential energy function V(x) does Ψ satisfy the schrodringer equation?
Answer:
What potential energy function gives this wave function? The function in question is a
gaussian, and if you read the book carefully you should have seen it already. Let’s not
spoil the fun though. If we know the wave function, and plug it into the time-
dependent Schrödinger’s equation, the only unknown should be the potential, so we
should be able to solve for it. First, we will need various derivatives of ψ.
Conveniently, for each of the derivatives you can factor out to simplify things.
∂ψ −2 am
= xψ
∂x ℏ
∂2 ψ −2 am 4 a2 m 2 2 −2 am 2 am 2
∂x 2
=

ψ+

x ψ=

1−

x ψ ( )
∂ψ
=−ia ψ
∂x
Substituting into the time-dependent Schrödinger equation, and solving for the V ψ
term:
∂ ψ −ℏ2 ∂2 ψ
iℏ = +V ψ
∂ x 2 m ∂ x2
∂ ψ ℏ 2 ∂2 ψ
V ψ=i ℏ +
∂ x 2 m ∂ x2
ℏ2 2am 2 am 2
V ψ=aℏψ −
2m ℏ
1−( ℏ
x ψ )
Now every term still has a left, so we can cancel all of those.
V =aℏ−aℏ+2 a2 m x 2 ⥤V =2 ma2 x 2
The potential that produces this wavefunction is a quadratic one, i.e., the simple
harmonic oscillator potential. Knowing this, we can make the usual identification that

1
V = k x 2 ,from which we would deduce k =4 ma 2 and ω= k =2 a for the given state
2 m √
of the oscillator.
c) Calculate the expectation values of x, x2 , p, p2.
Answer:
How about the expectation values? You can quickly convince yourself that

|⟨ x ⟩=0 , since x|ψ|2| is an odd function of x. You could also just asset this to be true -
since the potential is symmetric, the expected position should be in the center, atx=0.
∂ψ dψ
 Remember, if you are not familiar with partial derivatives, just read as .
∂t dt
2
∞ ∞ −2 am x
2 2 ℏ
⟨ x ⟩= ∫ x|ψ| dx= ∫ x A e dx=0
−∞ −∞

The same will hold for ⟨ p ⟩, though again you could assert that the particle must spend
as much time going one way as the other in a symmetric potential.
2 2
∞ −a ( ∞
[ ) ]ℏ
mx
−a (
[ ) ] dx
mx

⟨ p⟩∫ ψ
−∞
ℏ ∂ψ
i ∂x
¿
dx= ∫ A e
−∞

−it

i (
−2 am

Ae

+it

)
2
∞ −2 am x
2 ℏ
¿ 2 iamA ∫ xe dx=0
−∞

There is no really clever way to do the other two by inspection, just grind it out.
2
∞ ∞ −2 am x 3 /2
√π ℏ ℏ
2
⟨ x ⟩= ∫ x |ψ| dx =∫ x
−∞
2 2

−∞
2 2
A e ℏ
dx= A2
2 ( )
2 am
=
4 am

The momentum operator is ( ℏi )( dxd ) ,you can either square that or use 2m times
d2
kinetic energy. Either way, it is −ℏ
2
( )
dx 2
.

∞ 2 ∞
⟨ p ⟩ ∫ ψ (−ℏ ) ∂ ψ2 dx=A 2 ℏ2 ∫ |ψ|2 2 am 1− 2 am x 2 dx
2 ¿ 2
( )
−∞ ∂x −∞ ℏ ℏ
∞ ∞
2 2
¿ 2 amℏ ∫ A 2|ψ| dx−4 a2 m2 ∫ A 2 x 2|ψ| note 1 st term is norm . ,2 nd is ⟨ x 2 ⟩
−∞ −∞


¿ 2 amℏ−4 a2 m2 =amℏ
4 am
d) Find σ x and σ p. Is their product consitens with the uncertainty principle?
Answer:
ℏ ℏ
σ x =√ ⟨ x2 ⟩− ⟨ x 2 ⟩ =
√ 4 am
−0=
4 am √
σ p=√ ⟨ p2 ⟩− ⟨ p2 ⟩ =√ am ℏ−0=√ am ℏ

σ x σ p=
2
In fact, gaussian wave functions satisfy the minimum uncertainty principle allowed.

You might also like